¿Cómo entender el Lagrangiano de Dirac?

Tengo algunas preguntas básicas sobre cómo interpretar los lagrangianos, comencemos con Dirac:

L = Ψ ¯ ( i γ m m metro ) Ψ ,

dónde Ψ es un Dirac-Spinor, metro es la masa, γ m es una matriz gamma y m es la derivada.

1) es metro ¿un vector o una matriz o un escalar? Siempre pensé que es un escalar, pero por alguna razón Ψ ¯ metro Ψ está permitido en el Lagrangiano, pero metro Ψ ¯ Ψ no lo es, ¡así que no puede ser un escalar! EDITAR m es un escalar y puede estar en cualquier parte de la ecuación de Dirac, ¡pero estos términos no son válidos para el modelo estándar!

2) Qué es exactamente lo que se viola para que metro Ψ ¯ Ψ ¿No se permite? ¿No es invariante bajo algo? EDITAR estos términos no son válidos para el modelo estándar, ¡porque no son invariantes de calibre!

3) Siempre pensé que un espinor de Dirac contiene todo lo posible para un fermión, los dos estados de espín para una partícula y los dos estados de espín para la antipartícula. ¿Es correcta esta suposición?

4) ¿Por qué necesitamos Ψ ¯ ? ¿Que representa? ¿Las partículas y las antipartículas intercambian lugares? ¿O es mi interpretación en 3) incorrecta y Ψ representa la partícula y Ψ ¯ y antipartícula.

5) La ecuación de Dirac describe un fermión masivo libre que se mueve a través del espacio y el tiempo, ¿la Ψ ¯ indicar una interacción?

Estaba tratando de entenderlo de wikipedia, pero fallé. Cualquier respuesta a cualquiera de las preguntas anteriores, será apreciada.

Quién dice eso Ψ ¯ Ψ no está permitido? metro es un escalar para que puedas ponerlo donde quieras.
Un ejercicio de teoría cuántica de campos establece lo siguiente: "Los siguientes términos no están permitidos en el Modelo Lagrangiano Estándar. Para cada término, explique brevemente por qué. 1) metro Ψ ¯ Ψ " y luego algunos términos lagrangianos más...
Creo que malinterpretaste el enunciado del ejercicio. @Javier tiene razón.
¿Dónde? ¿En que contexto? ¿Dónde tal vez están hablando de invariancia de calibre?
@Alex Eso es algo completamente diferente. Para un fermión de Dirac genérico puedes escribir metro Ψ ¯ Ψ . En el modelo estándar no se puede porque viola la invariancia de calibre. Tenga en cuenta que no puede escribir Ψ ¯ metro Ψ ya sea por la misma razón.
@knzhou Pensé que la ecuación de Dirac es parte del modelo estándar. ¿Qué estaría permitido en el modelo Estándar?
El modelo estándar presenta campos de fermiones sin masa que obtienen una "masa efectiva" al interactuar con el campo de Higgs.
@ J.Murray Entonces, ¿básicamente cada término con una masa violaría la invariancia de calibre y, por lo tanto, no sería parte del modelo estándar?
Sí, eso es correcto.
@ J.Murray ¿Es la ecuación de Dirac parte de la teoría electrodébil, o estoy mezclando cosas aquí?
Si estás tomando un curso sobre física de partículas, te recomiendo que consigas un buen libro, como Introducción a las partículas elementales de Griffiths. Explica todo con claridad. Aprender este material de Wikipedia es realmente difícil, porque muchos artículos usan convenciones contradictorias, y algunos simplemente están equivocados.
Es posible que esté mezclando las ideas de la mecánica cuántica de partículas pseudo-relativistas con la teoría cuántica de campos realmente relativista.
No estoy tomando un curso, solo estoy tratando de aprender solo... Gracias a todos, eso responde al menos algunas de mis preguntas, ¿quizás también saben las respuestas a las otras?

Respuestas (1)

Por cierto, esa no es la ecuación de Dirac, sino el Lagrangiano/acción de Dirac.

1) metro es un escalar. Los términos de masa para los campos fermiónicos están permitidos en el modelo estándar, está confundiendo los términos de masa para los campos de calibre que no están permitidos por sí solos, sino que se producen a través de la ruptura espontánea de la simetría (mecanismo de Higgs).

2) metro Ψ Ψ ¯ está permitido, ya que cualquier cambio de fase (local de global) se cancelará. Nuevamente, está confundiendo un término de masa para un campo de calibre metro A m A m . Esto violaría la invariancia de calibre, A m A m + m Λ .

EDITAR Las dos respuestas anteriores son verdaderas para las interacciones de Dirac Lagrangians y EM, como se indica en la pregunta. En presencia de una interacción débil, los fermiones se ven afectados de manera diferente según su quiralidad. Esto luego introduce un término de masa dependiente del calibre, solo salvado por el mecanismo de Higgs.

3) No, es solo la ecuación de movimiento para un fermión de espín-1/2. Si construyes el operador de giro S 2 , encontrará que los valores propios son 3 4 2 , correspondiente a S ( S + 1 ) con S = 1 / 2 .

Para fermiones de espín 3/2, la ecuación es esta , etc.

4) ¿Cuál es la interpretación del complejo conjugado de un número? En realidad, simplemente inventa cualquier término en el Lagrangiano que le dé la ecuación de Dirac correcta (al aplicar las ecuaciones de Euler-Lagrange), que sabe que es correcta a partir del experimento.
Siempre puedes justificar la forma del Lagrangiano, por ejemplo teniendo Ψ Ψ ¯ significa que tiene invariancia de fase local y global, y que el potencial resultante Ψ 2 tiene un mínimo, lo que conduce a una teoría de campo estable.

5) Ψ ¯ no es una interacción. La ecuación de Dirac es la ecuación obedecida por un fermión de espín-1/2 masivo libre. O, más correctamente, por su operador de campo (por lo que hago la distinción entre el mecanismo cuántico relativista y la teoría cuántica de campos).

NB, sin embargo, puede establecer la masa en 0 , y obtienes los llamados fermiones de Weyl .

Para obtener interacciones, necesita términos no lineales.

El que suele salir es j m A m = Ψ γ m Ψ ¯ A m , dónde A m es el potencial de calibre electromagnético. Este término no es lineal y representa la interacción entre un fermión de espín-1/2 Ψ y un bosón vectorial spin-1 A m .

También puedes hacer que dos fermiones diferentes interactúen con un término que diga así Ψ 1 Ψ 2 , donde ambos obedecen a su ecuación de Dirac individual.


Wikipedia es realmente mala para estas cosas a menos que ya sepa aproximadamente lo que está sucediendo, recomendaría buscar cualquier serie de conferencias de pregrado sobre teorías de campo de calibre. El de Cambridge es bastante bueno.

¡Gracias por su respuesta! Estaba leyendo más y estoy un poco confundido por algunas declaraciones: 1) + 2) Esta fue una discusión en los comentarios, pero en realidad creo que los términos masivos no están permitidos en el SM, a menos que contengan el Escalar de Higgs. Este término no lo contiene, además no es calibre invariante, lo cual es un requisito para SM.
3) Creo que respondiste una pregunta diferente aquí. Ψ es un espinor, no una ecuación. Mis preguntas se referían a las entradas del espinor. 4) Ψ ¯ no es el conjugado complejo, es el conjugado hermitiano multiplicado por el γ 0 matriz Ψ ¯ = Ψ γ 0 , y no entiendo este objeto, pero entiendo el argumento de un objeto de maquillaje para que la ecuación funcione... En este campo, la carga conjugada parece tener la notación Ψ . 5) Así que aunque tenemos dos Ψ s en el Lagrangiano, es solo un fermión interactuando con el fotón A m ?
1) + 2), lo siento, pero no. Cualquier término de masa para ψ no violaría la invariancia de calibre. No sé su nivel y cuánto ya sabe, pero la invariancia de "calibre" aquí solo significa invariancia de "fase", y es trivial ver que metro Ψ ¯ Ψ se deja invariable por cualquier transformación de fase global o local, en términos de cualquier tu ( norte ) . Ahora, si comenzó con el lagrangiano de Dirac solo cinético + el potencial de Higgs, entonces seguro que el término de masa desaparecería naturalmente, con el metro siendo el coeficiente en términos del acoplamiento de Higgs gramo y el VVE.
Pero nada te impide incluir el metro Ψ ¯ Ψ Para empezar, es mejor obtenerlo del potencial de Higgs. Sin embargo, lo que no se le permite hacer es poner un término de masa para un campo de indicador de espín 1 A m , y metro A m A m no es invariante bajo la transformación de norma A m A m + m Λ como puedes demostrar trivialmente. Sin embargo, el hecho de que los bosones de calibre débil tengan masa significa que necesitamos el mecanismo de Higgs aquí para obtener el término de masa.
3) Entendí mal tu pregunta entonces, te pido disculpas. Tiene 4 grados de libertad, interpretados como espín ARRIBA y ABAJO, electrón y positrón.
4) Sé lo que es el adjunto Ψ ¯ Es decir, estaba tratando de hacer una analogía con una situación más simple. En la mecánica cuántica (no relativista), la función de onda ψ tiene una carga conservada | ψ | 2 , que sabes (supongo) se relaciona con la densidad de probabilidad. como te formas | ψ | 2 de ψ ? Con ψ , de ahí mi punto sobre el significado físico: es un requisito matemático. El adjunto viene por razones similares, usted sabe que la carga conservada aquí es la densidad de carga y la corriente de carga, cont--
--> entonces construyes tu invariante para darte estas respuestas, de ahí la introducción del factor de γ 0 que en realidad no sirve para ningún otro propósito ya que Ψ Ψ también le daría un escalar.
5) No entiendo tu comentario. Las interacciones solo surgen cuando tienes términos no lineales entre diferentes campos (espinor, vector o lo que quieras). Ψ ¯ no es un campo diferente al Ψ , es lo mismo, entonces Ψ ¯ Ψ no cuenta como un término de interacción.
(No estoy de acuerdo con algunas cosas en los comentarios a su pregunta, por lo que no quiero hacer referencia a ninguna de ellas; creo que algunos puntos solo aumentan su confusión).
Probablemente debería agregar, para completar, ya que se me acaba de ocurrir, que en el caso de los lagrangianos que no son de Dirac, entonces es cierto que metro Ψ ¯ Ψ no es invariante de calibre a menos que provenga del mecanismo de Higgs. Sin embargo, esto solo es cierto en el contexto de la interacción débil, ya que lo que atornilla la invariancia del indicador es el operador de paridad construido con γ 5 . Agregué una edición a mi respuesta.
(no paridad, quise decir quiralidad).